Care of the Adult Patient



Care of the Adult Patient





QUESTIONS

Each of the following questions or incomplete statements is followed by four or five suggested answers or completions. Select the ONE BEST ANSWER in each case.



1. All of the following can be helpful in preventing recurrent diverticulitis except


A) Increased dietary fiber


B) Exercise


C) Smoking cessation


D) Avoiding nuts, corn, or popcorn


E) Mesalamine and Lactobacillus casei

View Answer

Answer and Discussion

The answer is D. Interventions to prevent recurrences of diverticulitis include increased intake of dietary fiber, exercise, and, in persons with a body mass index (BMI) of 30 kg per m2 or higher, weight loss. Counseling for smoking cessation is recommended because smoking is associated with an increased incidence of complicated diverticulitis and less favorable outcomes (e.g., surgery at a younger age, higher risk of recurrence). Evidence from a prospective cohort study of 47,228 men in the United States found no evidence that avoiding nuts, corn, or popcorn decreases the risk of diverticulosis or diverticular complications, such as diverticulitis. A small prospective study found that mesalamine and Lactobacillus casei are effective in preventing recurrence. A meta-analysis of four randomized controlled trials with 1,660 patients who had experienced at least one episode of diverticulitis found that rifaximin (Xifaxan) plus fiber provided 1 year of complete relief and fewer complications compared with fiber alone.

Additional Reading: Diagnosis and management of acute diverticulitis. Am Fam Physician. 2013; 87(9):612-620.

Category: Gastroenterology system



2. Which one of the following represents an optimal screening strategy for colorectal cancer?


A) Colonoscopy every 5 years


B) Computed tomographic (CT) colonography every 10 years


C) High-sensitivity fecal occult blood test (FOBT) every 2 years


D) Sigmoidoscopy every 5 years with high-sensitivity FOBT every 3 years


E) Sigmoidoscopy every 5 years

View Answer

Answer and Discussion

The answer is D. The U.S. Preventive Services Task Force (USPSTF) recommends screening for colorectal cancer in adults 50 to 75 years of age using colonoscopy, sigmoidoscopy, or high-sensitivity FOBT. Studies show that the optimal intervals for these tests are colonoscopy every 10 years; high-sensitivity FOBT annually; and sigmoidoscopy every 5 years combined with high-sensitivity FOBT every 3 years. Sigmoidoscopy every 5 years without high-sensitivity FOBT is significantly less effective in detecting colorectal cancer than are other screening tests. The USPSTF concluded that there is insufficient evidence to determine the net benefit of CT colonography and fecal DNA testing.

Additional Reading: Screening for colorectal cancer. Am Fam Physician. 2010; 81(8):1017-1018.

Category: Gastroenterology system



3. Which of the following p values reflects the best chance that the findings are not the result of chance?


A) p value < 0.5


B) p value < 0.1


C) p value < 0.05


D) p value < 0.01


E) p value = 1

View Answer

Answer and Discussion

The answer is B. The p value is defined as the measured probability of a finding occurring (i.e., rejecting the null hypothesis) by chance alone, given that the null hypothesis is actually true. By convention, a p value < 0.05 is often considered significant. (“There is less than a 5% probability that the finding [null hypothesis rejected] was due to chance alone.”)

Additional Reading: Terms used in evidence-based medicine. Am Fam Physician. From: http://www.aafp.org/journals/afp/authors/ebm-toolkit/glossary.html.

Category: Patient/Population-based care




4. Which of the following statements is true regarding influenza?


A) Treatment with antivirals should be initiated within 48 hours of the onset of symptoms.


B) Anitiviral agents reduce the duration of fever by 1 week.


C) Amantadine is effective for both influenza types A and B.


D) Prophylactic therapy is the single most important measure to prevent influenza outbreaks.


E) Amantadine is the only agent approved for prophylaxis.

View Answer

Answer and Discussion

The answer is A. Influenza causes significant morbidity and mortality and is responsible for considerable medical expenditures especially in the elderly. Vaccination is the single most important public health measure to prevent this illness. Amantadine (Symmetrel) and rimantadine (Flumadine) are older antiviral agents (M2 inhibitors). Oseltamivir (Tamiflu) and zanamivir (Relenza) are chemically related antiviral medications known as neuraminidase inhibitors that have activity against both influenza A and B viruses. Both are also approved for prophylaxis. For antiviral agents to be effective, they must be initiated within 48 hours of the onset of influenza symptoms. Antiviral agents reduce the duration of fever and illness by 1 day and also reduce the severity of some symptoms. For optimal use of antiviral agents, patients with influenza symptoms must present early, and family physicians must accurately and rapidly diagnose the illness.

Additional Reading: Influenza antiviral medications. Centers for disease control and prevention. From: http://www.cdc.gov/flu/professionals/antivirals/summary-clinicians.htm. Accessed 8/22/2013.

Category: Respiratory system



5. Primary insomnia is usually associated with


A) Sleep apnea


B) Restless legs syndrome (RLS)


C) Periodic limb movements


D) Circadian rhythm sleep disorders


E) None of the above

View Answer

Answer and Discussion

The answer is E. Insomnia is defined as inadequate or poor-quality sleep characterized by one or more of the following: difficulty falling asleep, difficulty maintaining sleep, waking up too early in the morning, or sleep that is not refreshing. Insomnia also involves daytime consequences such as fatigue, difficulty concentrating, and irritability. Periods of insomnia lasting between 1 night and a few weeks are defined as acute insomnia. Chronic insomnia refers to sleep difficulty occurring at least 3 nights per week for 1 month or more. Insomnia may be associated with specific sleep disorders, including RLS, periodic limb movement disorder, sleep apnea, and circadian rhythm sleep disorders.

RLS is characterized by unpleasant sensations in the legs or feet that are temporarily relieved by movement. Symptoms are worse in the evening, especially when a person is lying down and remaining still. The sensations cause difficulty falling asleep and are often accompanied by periodic limb movements. Periodic limb movement disorder is characterized by bilateral repeated and rhythmic, smallamplitude jerking or twitching movements in the lower extremities and, less frequently, in the arms. These movements occur every 20 to 90 seconds and can lead to awakenings, which are usually not noticed by the patient. Often the patient reports that sleep is not refreshing. In many cases, the bed partner is more likely to report the movement problem.

Obstructive sleep apnea is most commonly associated with snoring, daytime sleepiness, and obesity but occasionally presents with insomnia. Circadian rhythm sleep disorders including sleepwork insomnia are characterized by an inability to sleep because of a disturbance between the circadian sleep rhythm and the desired or required sleep schedule.

Primary insomnia occurs in the absence of the previously mentioned conditions. When the insomnia persists beyond 1 or 2 nights or becomes predictable, treatment should be considered. Pharmacologic treatment is usually effective, especially short-acting hypnotics. Sleep hygiene measures may also be useful. Chronic insomnia may be more difficult to treat. Because chronic insomnia is often multifactorial in etiology, a patient may need multiple treatment modalities, including medication (antidepressants, antihistamines, melatonin) and behavioral therapy. If an underlying medical or psychiatric condition is identified, this condition should be treated first.

Additional Reading: Primary insomnia in older persons. Am Fam Physician. 2013; 87(4):280-281.

Category: Neurology



6. A 65-year-old man presents to your office and reports urinary incontinence. Examination reveals an enlarged prostate. You suspect overflow incontinence. Which of the following measurements of postvoid residual (PVR) volumes would represent the threshold for a normal amount?


A) 50 mL


B) 100 mL


C) 200 mL


D) 500 mL


E) 1,000 mL

View Answer

Answer and Discussion

The answer is A. Urge incontinence results from bladder contractions that exceed the ability of the brain to prevent them. Causes include inflammation or irritation within the bladder, resulting from calculi, malignancy, infection, or atrophic vaginitis-urethritis. Other central causes include stroke, Parkinson’s disease or dementia, drugs such as hypnotics or narcotics, or metabolic disorders such as hypoxemia and encephalopathy. Additionally, urge incontinence can occur when ambulation is impaired, making it difficult for patients to get to the bathroom in time. This condition is referred to as “functional incontinence.”

Stress incontinence is caused by a malfunction of the urethral sphincter that causes urine to leak from the bladder when intraabdominal pressure increases, such as during coughing or sneezing. Causes for stress incontinence include pelvic prolapse, urethral hypermobility, or displacement of the urethra and bladder neck from their normal anatomic alignment. Stress incontinence can also occur as a result of intrinsic sphincter deficiency, in which the sphincter is weak because of a congenital condition or denervation resulting from α-adrenergic blocking drugs, surgical trauma, or radiation damage. Overflow bladder incontinence occurs as a result of urine retention with bladder distention. Urine collects in the bladder until maximum bladder capacity is reached. It then leaks as a result of “overflow,” usually manifesting as dribbling. Increased intra-abdominal pressure may also cause loss of urine, so that overflow incontinence sometimes mimics stress incontinence.

Overflow incontinence can be caused by medications that relax the bladder detrusor muscle (e.g., anticholinergic agents, calciumchannel blockers [CCBs]). It can also be caused by denervation of the detrusor resulting from a neurologic abnormality that affects
bladder innervation (e.g., diabetic neuropathy) or because of damage to bladder innervation (e.g., tumors, radiation, surgery). Additionally, overflow incontinence can be caused by obstructed urinary outflow resulting from prostate enlargement, fecal impaction, urethral stricture, or urethral constriction related to α-adrenergic agonist medications.

The workup should include a urinalysis, which can identify acute urinary tract infection (UTI) and diabetes-induced glycosuria, both of which can cause or aggravate urge incontinence. These conditions are reversible with treatment. If appropriate, a urine culture should also be obtained. The basic evaluation of urinary incontinence should also include measurement of PVR urine volume to detect urinary retention (i.e., overflow bladder). PVR volume measurement can detect retention caused by potentially reversible factors (e.g., anticholinergic or other drugs, fecal impaction). Urinary retention not obviously resulting from a transient cause generally requires further evaluation, including cystometry, to determine why the bladder does not empty properly. The PVR urine volume can be measured by one of two methods. The first and most common method is “in and out” urethral catheterization after the patient has urinated to empty the bladder. The quantity of urine obtained is measured. PVR volume can also be measured with pelvic ultrasonography. Ultrasonography is a useful alternative to catheterization, especially for measuring the PVR volume in men with suspected prostate obstruction, because catheterizing these patients may cause urinary infection or obstruction. Normally, <50 mL of residual urine is present after voiding. Volumes of >200 mL are abnormal. Intermediate volumes (50 to 200 mL) are considered equivocal, and the test should be repeated. Other tests include office cystometry and office stress testing. Office cystometry consists of aliquots of sterile saline that are infused into the bladder via a catheter with an open syringe attached to the catheter. Contractions are detected by monitoring the fluid level that appears in the syringe after several aliquots of water have been instilled. A rise and fall in the fluid level indicates pressure changes (i.e., contractions) within the bladder. Severe feelings of urgency or bladder contractions at <300 mL of bladder volume constitute a presumptive diagnosis of urge incontinence. For the diagnosis of urge incontinence, simple cystometry can be used. In this test, the patient lies supine on the examination table with a full bladder and coughs forcefully. The physician places a gauze pad in front of the perineum. If urine leaks onto the gauze pad during coughing, a presumptive diagnosis of stress incontinence is made. The physician then places his or her fingers on either side of the patient’s urethra and elevates the structure. The patient is then asked to cough. In patients with stress incontinence, urethral elevation prevents further urine leakage. If no incontinence is noted in the supine position, the maneuvers should be repeated with the patient standing. If no incontinence occurs in either position, the patient probably does not have stress incontinence.

Additional Reading: Diagnosis of urinary incontinence. Am Fam Physician. 2013; 87(8):543-550.

Category: Nephrologic system



7. Which of the following is considered a risk factor for retinal detachment?


A) Glaucoma


B) Diabetic retinopathy


C) Hyphema


D) Myopia

View Answer

Answer and Discussion

The answer is D. Retinal detachment is a preventable cause of vision loss. It is relatively common after the age of 60. There are three types of retinal detachments: exudative, tractional, and rhegmatogenous. The most common type is rhegmatogenous, which results from retinal breaks caused by vitreoretinal traction. Exudative (or serous) retinal detachment results from the accumulation of serous and/or hemorrhagic fluid in the subretinal space because of hydrostatic factors (e.g., severe acute hypertension), or inflammation (e.g., sarcoid uveitis), or neoplastic effusions. Exudative retinal detachment generally resolves with adequate treatment of the underlying disease, and restoration of normal vision is often excellent. Tractional retinal detachment occurs via centripetal mechanical forces on the retina, usually mediated by fibrotic tissue resulting from previous hemorrhage, injury, surgery, infection, or inflammation. Risk factors for retinal detachment include advancing age, previous cataract surgery, myopia, and trauma. Other eye conditions including hyphema, glaucoma, and diabetic retinopathy are not considered risk factors for retinal detachment. Patients typically present with symptoms such as light flashes, floaters, peripheral visual field loss, and blurred vision. Retinal tears may occur without symptoms, but often photopsia (light flashes) is noted. Photopsia results from vitreoretinal traction. When the retina tears, blood and retinal pigment epithelium cells may enter the vitreous cavity and are perceived as “floaters.” Immediate intervention can prevent retinal detachment. Patients with the acute onset of flashes or floaters should be referred to an ophthalmologist.

Additional Reading: Evaluation and management of suspected retinal detachment. Am Fam Physician. 2004;69:1691-1698.

Category: Special sensory



8. Which of the following medications is considered the treatment of choice for Bordetella pertussis infection?


A) Penicillin


B) Ciprofloxacin


C) Azithromycin


D) Tetracycline


E) Cefuroxime

View Answer

Answer and Discussion

The answer is C. Recent epidemiologic studies have shown that the incidence and prevalence of Bordetella pertussis infection in adults are much greater than previously reported. In studies of adults with chronic cough, 20% to 25% were found to have serologic evidence of recent B. pertussis infection. However, pertussis is rarely considered in adults because the signs and symptoms are nonspecific. Apart from a prolonged cough, there are no specific symptoms suggestive of pertussis in older individuals who have been immunized. With this in mind, pertussis should be considered in the differential diagnosis of persistent cough in previously immunized children and adults. Administration of erythromycin or other macrolide (azithromycin or clarithromycin) may be a consideration in patients presenting with persistent cough. Prophylaxis of exposed persons before culture or serologic results are available would be another consideration. Early treatment with a macrolide should limit the spread of infection to persons whose immunity has waned or in unimmunized children. The acellular vaccine may allow booster immunization, which can be a method of preventing B. pertussis infection after immunity from the pertussis vaccination has waned.

Additional Reading: Prevention and treatment of pertussis. Med Lett Drugs Ther. 2012;54(1399):73-74.

Category: Respiratory system




9. Mrs Jones, a 52-year-old teacher, presents with a sudden onset of dyspnea. Which one of the following makes a pulmonary embolus more likely?


A) Fever >38.0°C (100.4°F)


B) Chest pain


C) Orthopnea


D) Wheezes


E) Rhonchi

View Answer

Answer and Discussion

The answer is B. Chest pain is common in patients with pulmonary embolism (PE). When evaluating a patient for possible PE, the presence of orthopnea suggests heart failure, fever suggests an infectious process, wheezing suggests asthma or chronic obstructive pulmonary disease (COPD), and rhonchi suggest heart failure, interstitial lung disease, or infection. These generalizations are supported by a 2008 study designed to improve the diagnosis of PE on the basis of the history, physical examination, EKG, and chest radiograph.

Additional Reading: Diagnosis of deep venous thrombosis and pulmonary embolism. Am Fam Physician. 2012;86(10):913-919

Category: Cardiovascular system



10. A 23-year-old auto mechanic is brought to the emergency department with slurred speech, confusion, and ataxia. He appears intoxicated and has a history of binge drinking, but denies recent alcohol intake and no odor of alcohol is noted on his breath. Abnormalities on the metabolic profile include a carbon dioxide content of 10 mmol per L (N 20 to 30). His blood alcohol level is <10 mg per dL (0.01%). A urinalysis shows calcium oxalate crystals and a red blood cell (RBC) count of 10 to 20 per hpf. Woods lamp examination of the urine shows fluorescence. His arterial pH is 7.25. Which one of the following would be the most appropriate at this point?


A) Immediate hemodialysis


B) Gastric lavage


C) Administration of activated charcoal


D) Fomepizole (Antizol)

View Answer

Answer and Discussion

The answer is D. Ethylene glycol poisoning should be suspected in patients with metabolic acidosis of unknown cause and subsequent renal failure, as rapid diagnosis and treatment will limit the toxicity and decrease both morbidity and mortality. This diagnosis should be considered in a patient who appears intoxicated but does not have an odor of alcohol, and has anion gap acidosis, hypocalcemia, urinary crystals, and nontoxic blood alcohol levels. Ethylene glycol is found in products such as engine coolant, de-icing solution, and carpet and fabric cleaners. Ingestion of 100 mL of ethylene glycol by an adult can result in toxicity.

Until recently, ethylene glycol poisoning was treated with sodium bicarbonate, ethanol, and hemodialysis. Treatment with fomepizole (Antizol) has this specific indication, however, and should be initiated immediately when ethylene glycol poisoning is suspected. If ethylene glycol poisoning is treated early, hemodialysis may be avoided, but once severe acidosis and renal failure have occurred hemodialysis is necessary. Ethylene glycol is rapidly absorbed, and use of ipecac or gastric lavage is therefore not effective. Large amounts of activated charcoal will only bind to relatively small amounts of ethylene glycol, and the therapeutic window for accomplishing this is less than 1 hour.

Additional Reading: Methanol and ethylene glycol poisoning. In: Basow DS, ed. UpToDate. Waltham, MA: UpToDate; 2013.

Category: Nonspecific system



11. The Health Insurance Portability and Accountability Act (HIPAA) standards ensure that


A) Patients have control and access to their medical records


B) Insurance companies have unlimited access to health information


C) Physicians can protect themselves from liability


D) Attorneys have unrestricted access to health care records

View Answer

Answer and Discussion

The answer is A. HIPAA is three sets of standards developed by the Department of Health and Human Services in 1996: (1) Transactions and Code Sets, (2) Privacy, and (3) Security. The goals of the standards are to simplify the administration of health insurance claims and lower costs, give patients more control and access to their medical information, and protect individually identifiable medical information from real or potential threats of disclosure or loss. Privacy and security are closely related. Privacy is the patient’s right over the use and disclosure of his or her own personal health information. Privacy includes the right to determine when, how, and to what extent personal information is shared with others. The HIPAA privacy rules grant new rights to patients to gain access to and control the use and disclosure of their personal health information. Security is the specific measures a health care provider must take to protect personal health information from unauthorized breaches of privacy, as in situations where information is stolen or sent to the wrong person in error. Security also includes measures taken to ensure against the loss of personal health information, as in situations where a patient’s records are lost or destroyed by accident. The HIPAA privacy rules require general security measures be put in place, and the proposed security rules follow a detailed and comprehensive set of activities to guard against unauthorized disclosure of personal health information stored or transmitted electronically or on paper.

Additional Reading: HIPAA again: confronting the updated privacy and security rules. Fam Pract Manag. 2013; 20(3):18-22.

Category: Patient/Population-based care



12. You are asked to perform a preoperative evaluation on a 55-yearold white woman with type 2 diabetes mellitus prior to elective femoral-anterior tibial artery bypass surgery. She is unable to climb a flight of stairs or do heavy work around the house. She denies exertional chest pain and is otherwise healthy. Based on current guidelines, which one of the following diagnostic studies would be appropriate prior to surgery because the results could alter the management of this patient?


A) Pulmonary function studies


B) Coronary angiography


C) Carotid angiography


D) A dipyridamole-thallium scan


E) A hemoglobin A1c level

View Answer

Answer and Discussion

The answer is D. A preoperative evaluation prior to noncardiac surgery requires an assessment of the perioperative cardiovascular risk of the procedure involved, the functional status of the patient, and clinical factors that can increase the risk, such as diabetes mellitus, stroke, renal insufficiency, compensated or prior heart failure, mild angina, or previous myocardial infarction (MI). This patient is not undergoing emergency surgery, nor does she have an active cardiac condition; however, she is undergoing a high-risk procedure (>5% risk of perioperative MI) with vascular surgery. As she cannot climb a flight of stairs or do heavy housework, her functional status is <4 METs, and she should be considered for further evaluation. The patient’s diabetes is an additional clinical risk factor.


With vascular surgery being planned, appropriate recommendations include proceeding with the surgery with heart rate control, or performing noninvasive testing if it will change the management of the patient. Coronary angiography is indicated if the noninvasive testing is abnormal. Pulmonary function studies are most useful in patients with underlying lung disease or those undergoing pulmonary resection. Hemoglobin A1c is a measure of long-term diabetic control and is not particularly useful perioperatively. Carotid angiography is not indicated in asymptomatic patients being considered for lower-extremity vascular procedures.

Additional Reading: Perioperative cardiac risk reduction. Am Fam Physician. 2012;85(3):239-246.

Category: Cardiovascular system



13. Which of the following tumor markers is correct for the condition?


A) CA (cancer antigen) 27.29 for metastatic cervical cancer


B) CA (cancer antigen) 125 for hepatic carcinoma


C) AFP (α-fetoprotein) for ovarian carcinoma


D) CA (cancer antigen) 19-9 for pancreatic cancer


E) β-hCG (beta unit of human chorionic gonadotropin) for ovarian cancer

View Answer

Answer and Discussion

The answer is D. Recognized tumor markers are most appropriate for monitoring response to therapy and detecting early recurrence. Cancer antigen (CA) 27.29 is most often used to follow response to therapy in patients with metastatic breast cancer. CA 27.29 is highly associated with breast cancer, although levels are elevated in several other malignancies (colon, gastric, hepatic, lung, pancreatic, ovarian, and prostate cancers). CA 27.29 also can be found in patients with benign disorders of the breast, liver, and kidney and in patients with ovarian cysts. CA 27.29 levels higher than 100 units per mL are rare in benign conditions.

Carcinoembryonic antigen (CEA) is used to detect relapse of colorectal cancer. CEA elevations also occur with other malignancies. Nonmalignant conditions associated with elevated CEA levels include cigarette smoking, peptic ulcer disease, inflammatory bowel disease, pancreatitis, hypothyroidism, biliary obstruction, and cirrhosis. Levels exceeding 10 ng per mL are rarely due to benign disease. Fewer than 25% of patients with disease confined to the colon have an elevated CEA level. Therefore, CEA is not useful in screening for colorectal cancer or in the diagnostic evaluation of an undiagnosed illness. A CEA level should be utilized only after malignancy has been diagnosed.

CA 19-9 may be helpful in diagnosing pancreatic abnormalities. Levels >1,000 units per mL are correlated with pancreatic cancer. Benign conditions such as cirrhosis, cholestasis, cholangitis, and pancreatitis can also result in CA 19-9 elevations, although values are usually <1,000 units per mL.

CA 125 is useful for evaluating pelvic masses in postmenopausal women, monitoring response to therapy in women with ovarian cancer, and detecting recurrence of ovarian carcinoma. Postmenopausal women with asymptomatic palpable pelvic masses and CA 125 levels >65 units per mL likely have ovarian cancer. Because premenopausal women have more benign causes of elevated CA 125 levels, testing for the marker is less useful in this population.

α-Fetoprotein (AFP) is a marker for hepatocellular carcinoma. It is used to screen highly selected populations and to assess hepatic masses in patients at particular risk for developing hepatic malignancy.

Testing for the β-subunit of human chorionic gonadotropin (β-hCG) is an integral part of the diagnosis and management of gestational trophoblastic disease. Combined AFP and β-hCG testing is an essential adjunct in the evaluation and treatment of nonseminomatous germ cell tumors and in monitoring the response to therapy. AFP and β-hCG are useful in evaluating potential origins of poorly differentiated metastatic cancer.

Prostate-specific antigen (PSA) is used to screen for prostate cancer and detects recurrence of the malignancy.

Additional Reading: Serum tumor markers. Am Fam Physician. 2003;68:1075-1082.

Category: Nonspecific system



14. Which one of the following is the greatest risk factor for abdominal aortic aneurysm (AAA)?


A) Cigarette smoking


B) Diabetes mellitus


C) Hypertension


D) African American race


E) Female gender

View Answer

Answer and Discussion

The answer is A. Cigarette smokers are five times more likely than nonsmokers to develop an AAA. The risk is associated with the number of years the patient has smoked and declines with cessation. Diabetes mellitus is protective, decreasing the risk of AAA by half. Women tend to develop AAA in their sixties, 10 years later than men. Whites are at greater risk than African Americans. Hypertension is less of a risk factor than cigarette smoking.

Additional Reading: The U.S. Preventive Services Task Force (USPSTF) recommendations on screening for abdominal aortic aneurysm. From: http://www.uspreventiveservicestaskforce.org/uspstf/uspsaneu.htm

Category: Cardiovascular system



15. A 23-year-old otherwise healthy white woman presents complaining of dizziness. She was fine until about a week ago when she developed a case of diarrhea, which other members of her family have also had. Since that time she has been lightheaded when standing, feels her heart race, and gets headaches or blurred vision if she does not sit or lie down. She has not passed out but has been unable to work. She has not taken any medications. On exam, no abnormalities are detected except for her heart rate, which increases from 72 beats per minute when she is sitting to 112 beats per minute when she stands. Her blood pressure remains unchanged with positional changes. Routine laboratory tests and an EKG are normal. What is the most likely cause of this patient’s condition?


A) Myocarditis


B) A seizure disorder


C) Postural orthostatic tachycardia syndrome (POTS)


D) Systemic lupus erythematosus (SLE)


E) Somatization disorder

View Answer

Answer and Discussion

The answer is C. POTS is manifested by a rise in heart rate >30 beats per minute or by a heart rate >120 beats per minute within 10 minutes of being in the upright position. Symptoms usually include positiondependent headaches, abdominal pain, light-headedness, palpitations, sweating, and nausea. Most patients will not actually pass out, but some will if they are unable to lie down quickly enough. This condition is most prevalent in white women between the ages of 15 and 50 years. Often these patients are hardworking, athletic, and otherwise in good health. There is a high clinical correlation between POTS and chronic fatigue syndrome. Although no single etiology for POTS has been found, the condition is thought to have a genetic predisposition, is often incited after a prolonged viral illness, and has a component of deconditioning. The recommended initial management is encouraging adequate fluid and salt intake, followed by the initiation of regular aerobic exercise combined with lower-extremity strength training, and then the use of β-blockers.


Additional Reading: Dizziness: a diagnostic approach. Am Fam Physician. 2010;82(4):361-368, 369; and Postural orthostatic tachycardia syndrome: a clinical review. Pediatr Neurol. 2010;42(2):77-85.

Category: Cardiovascular system



16. Of the following conditions, which is related to the development of osteoporosis in men?


A) Prolactinoma


B) Hypogonadism


C) Prostate cancer


D) Renal stones


E) Inguinal hernia

View Answer

Answer and Discussion

The answer is B. Men, like women, are at risk of developing osteoporosis that may lead to increased risk of fractures. Hypogonadism is an independent risk factor for osteoporosis. Dual-energy x-ray absorptiometry should be performed in men who are at increased risk of osteoporosis and who are candidates for drug therapy. Initial labs in men with osteoporosis should include a complete blood count, liver function testing, thyroid-stimulating hormone (TSH) level, serum testosterone, creatinine, calcium, and 25-OH vitamin D levels. Twenty-four-hour urine calcium and creatinine levels to identify hypercalciuria is indicated in men with osteoporosis that occurs before the age of 60 years or if initial diagnostic methods fail to determine a cause of low bone mass.

Bisphosphonates decrease the risk of vertebral fracture in men with osteoporosis. Teriparatide (Forteo) decreases the risk of vertebral fractures and can be used for treatment of severe osteoporosis. Men should have an adequate intake of calcium (1,200 mg daily) along with vitamin D (800 IU daily) to prevent osteoporosis.

Additional Reading: Osteoporosis in men. Am Fam Physician. 2010;82(5):503-508.

Category: Endocrine system



17. In patients treated with disulfiram (Antabuse) for alcohol abuse, which test is necessary for monitoring during treatment?


A) Alkaline phosphatase


B) Amylase


C) Creatinine


D) Alanine aminotransferase (ALT)


E) Ammonia level

View Answer

Answer and Discussion

The answer is D. In the United States, disulfiram, acamprosate (Campral), and naltrexone (Revia) are approved for the treatment of alcohol dependence. Although disulfiram is reported to be effective as an aversive drug, placebo-controlled clinical trials have been inconclusive. Disulfiram inhibits the metabolism of anticoagulant drugs, phenytoin, and isoniazid. It should be used cautiously in patients with liver disease and is contraindicated during pregnancy and in patients with ischemic heart disease. Disulfiram can cause hepatitis, and therefore monitoring of liver function studies is essential. Acamprosate is a newer agent that works by its effect on the GABA system. Side effects include diarrhea, insomnia, anxiety, depression, pruritus, and dizziness. The third drug approved for use in the treatment of alcohol dependence is the opioid antagonist naltrexone. Naltrexone is believed to reduce consumption of alcohol and increase abstinence by reducing the craving for alcohol. The rate of relapse is highest within the first 90 days of abstinence, and it is during this time that naltrexone may be beneficial. Daily dosages may range from 25 to 100 mg. Side effects include nausea, headache, anxiety, and sedation. Naltrexone can be hepatotoxic at higher dosages and should be used with caution in patients with chronic liver disease. Selective serotonin reuptake inhibitors (SSRIs), including fluoxetine and sertraline, have been found to decrease alcohol intake in heavy drinkers without a history of depression. However, in some trials SSRIs were found to be no more effective than a placebo. Any drug therapy should be combined with psychotherapy or group therapy to help address the social and psychologic aspects of alcohol dependence.

Additional Reading: Management of alcohol withdrawal syndrome. Am Fam Physician. 2010;82(4):344-347.

Category: Psychogenic




18. The test of choice for the diagnosis of ureteral obstruction secondary to renal lithiasis is


A) Noncontrast helical computed tomography (CT)


B) Ultrasound


C) Intravenous pyelogram


D) Magnetic resonance imaging (MRI)


E) Plain radiographs

View Answer

Answer and Discussion

The answer is A. The most common cause of the sudden onset of flank pain in adults is acute urolithiasis. Identification of a stone in the ureter with resultant partial or complete ureteral obstruction confirms the suspected diagnosis. Noncontrast helical CT has the advantages of avoiding contrast exposure, identifying radiolucent calculi, evaluating nearby structures and requires a shorter time for examination. As a result, helical CT is a better test for assessing acute urolithiasis. Abdominal ultrasound has a high specificity in evaluating stones, but its sensitivity is lower than helical CT. Plain films are used to follow patients with known radiopaque (i.e., calcium) stones.

Additional Reading: Guidelines on urolithiasis. European association of urology (EAU). 2008; 128 p.

Category: Nephrologic



19. Which of the following infections is least likely in a patient with COPD?


A) Streptococcus pneumoniae


B) Haemophilus influenzae


C) Moraxella catarrhalis


D) Mycoplasma pneumoniae

View Answer

Answer and Discussion

The answer is D. COPD is a disease process involving progressive chronic airflow obstruction because of chronic bronchitis, emphysema, or both. Chronic bronchitis is defined clinically as excessive cough and sputum production on most days for at least 3 months during at least 2 consecutive years. Emphysema is characterized by chronic dyspnea resulting from the destruction of lung tissue and the enlargement of air spaces. Asthma, which features airflow obstruction, airway inflammation, and increased airway responsiveness to various stimuli, may be distinguished from COPD by reversibility of pulmonary function deficits. Acute exacerbations of COPD are
treated with oxygen (in hypoxemic patients), inhaled β2 agonists, inhaled anticholinergics, antibiotics, and systemic corticosteroids. Theophylline may be considered in patients who do not respond to other bronchodilators.

Antibiotic therapy is directed at the most common pathogens, including Streptococcus pneumoniae, Haemophilus influenzae, and Moraxella catarrhalis. Mild-to-moderate exacerbations of COPD are usually treated with broad-spectrum antibiotics such as doxycycline, trimethoprim-sulfamethoxazole, and amoxicillin-clavulanate potassium. Treatment with extended-spectrum penicillins, fluoroquinolones, third-generation cephalosporins, or aminoglycosides may be considered in patients with severe exacerbations. The management of chronic stable COPD includes smoking cessation and oxygen therapy. Inhaled β2 agonists, inhaled anticholinergics, and systemic corticosteroids are also used in patients with chronic stable disease. Inhaled corticosteroids decrease airway reactivity and can reduce the use of health care services for management of respiratory symptoms. Avoiding acute exacerbations helps to reduce long-term complications. Long-term oxygen therapy, regular monitoring of pulmonary function, and referral for pulmonary rehabilitation are often utilized and can improve the quality of life and reduce hospitalizations. Influenza and pneumococcal vaccines should be administered. Selected patients who do not respond to standard therapies may benefit from lung-reduction surgery.

Additional Reading: ACP updates guideline on diagnosis and management of stable COPD. Am Fam Physician. 2012;85(2): 204-205.

Category: Respiratory system



20. Because of safety concerns, which one of the following asthma medications should be used only as additive therapy and not as monotherapy?


A) Inhaled corticosteroids


B) Leukotriene-receptor antagonists


C) Short-acting β-agonists


D) Long-acting β-agonists


E) Mast-cell stabilizers

View Answer

Answer and Discussion

The answer is D. Because of the risk of asthma exacerbation or asthma-related death, the Food and Drug Administration (FDA) has added a warning against the use of long-acting β-agonists as monotherapy. Inhaled corticosteroids, leukotriene-receptor antagonists, short-acting β-agonists, and mast-cell stabilizers are approved and accepted for both monotherapy and combination therapy in the management of asthma.

Additional Reading: Drugs for asthma and COPD. Treat Guidel Med Lett. 2013;132:75.

Category: Respiratory system



21. A 65-year-old with a history of chronic atrial fibrillation is being monitored while on warfarin therapy. The nurse calls to inform you the patient’s international normalized ratio (INR) is measured at 7. He has no active signs of bleeding, but is at increased risk of bleeding. Appropriate management at this time includes which of the following:


A) Stop warfarin, observe, and repeat INR in 3 days.


B) Stop warfarin and observe; repeat INR in 24 hours.


C) Stop warfarin, give vitamin K, and repeat INR in 24 hours.


D) Stop warfarin, give vitamin K and fresh-frozen plasma with daily INRs.

View Answer

Answer and Discussion

The answer is C. Warfarin inhibits the formation of clotting factors II, VII, IX, and X. The drug is highly protein-bound to albumin. Because of this inverse relationship between the levels of albumin and free warfarin, acutely ill patients with poor nutritional states and postoperative patients may need lower dosages of warfarin. The INR is the patient’s prothrombin time divided by the mean of the normal prothrombin time, with this ratio raised to the international sensitivity index. After starting warfarin therapy, a steadystate response is not typically achieved for approximately 2 weeks. A dosage of 4 to 5 mg per day is typical, although the required dosage may be variable (as low as 0.5 mg or as high as 50 mg per day). Elderly patients should start at a lower dosage. Checking the INR approximately 24 hours after the first dose can help determine the second dose. If there has been little or no rise in the INR (which is to be expected), a 5-mg dose on the second day should be safe. If an INR is not available on the day after the first dose, it can be obtained on days 2, 3, or 4. If the initial INR (on days 1 through 4) is high, the patient is likely sensitive to warfarin’s effects; therefore, a lower dose should be given. Patients who are restarting warfarin therapy after a time off the drug can safely begin with their previous maintenance dose. Guidelines recommend that the INR be checked at least four times during the first week of therapy. This frequency could then be gradually decreased on the basis of the stability of the INR. Because the risk of bleeding is greatest in the first 6 to 12 weeks of treatment, checking the INR weekly is appropriate. The maximum time between tests should be no more than 4 to 6 weeks. If a patient’s INR has been stable and then fluctuates by more than 0.2 below or 0.4 above the goal INR, the patient should be evaluated for the cause of the change. Associated causes include laboratory error, noncompliance, drug interactions with warfarin, dietary interactions, or a change in the patient’s health. If no reversible cause is found, a change in dosage may be made, with a repeat INR within 2 weeks. Close follow-up with repeated testing is needed because the patients who have the most variation in results are most likely to develop bleeding or thromboembolism. In asymptomatic patients whose INR is elevated, temporary discontinuation of the drug is often used, but administration of vitamin K shortens the time to return to the target INR. There is indirect evidence that use of vitamin K is associated with a lower incidence of hemorrhage. Oral vitamin K is effective and may have fewer risks than the parenterally administered form. When a patient’s INR is between 5 and 9, the recommendations include temporary discontinuation of warfarin therapy. If the patient is at risk for hemorrhage (e.g., is taking nonsteroidal anti-inflammatory drugs [NSAIDs]), low-dose oral vitamin K (1.0 to 2.5 mg) also should be given. However, the lowest dose available in tablet form is 5 mg, and often only 1 or 2 mg is needed. The parenteral form can be given orally and mixed in a flavored drink if needed. If the patient cannot be treated orally, 0.5 to 1 mg of intravenous vitamin K should be administered. For INRs of 9 or higher, vitamin K also should be given at a higher dose (2.5 mg intravenously or 5 mg orally). A repeat INR should be obtained within 24 hours. Additional vitamin K may be needed, depending on the result of the repeat INR. If the INR is elevated and the patient is bleeding, fresh-frozen plasma or a concentrate of clotting factors should be administered. A repeat INR should be obtained shortly after the fresh-frozen plasma is given. Additional fresh-frozen plasma may be needed because of its short duration of action. A large dose of vitamin K (10 mg) should also be given. Additional vitamin K may be necessary because the halflife of warfarin is longer than the half-life of vitamin K. Daily INR measurements should be instituted.


Additional Reading: A systematic approach to managing warfarin doses. Fam Pract Manag. 2005;12(5):77-83.

Category: Cardiovascular system



22. A 60-year-old man presents to your office inquiring about prostate cancer screening. Choose the correct statement:


A) PSA is the gold standard test for prostate cancer screening.


B) PSA should be checked annually starting at 50 years of age to screen for prostate cancer.


C) PSA can produce false-positive results, which can be associated with negative psychological effects.


D) Men who have false-positive test are less likely to have additional testing.


E) The USPSTF recommends PSA testing to screen for prostate cancer at age 60.

View Answer

Answer and Discussion

The answer is C. Convincing evidence demonstrates that the PSA test often produces false-positive results; approximately 80 percent of positive PSA test results are false-positive when cutoffs between 2.5 and 4.0 ng per mL (2.5 and 4.0 µg per L) are used. There is adequate evidence that false-positive PSA test results are associated with negative psychological effects, including persistent worry about prostate cancer. Men who have a false-positive test result are more likely to have additional testing, including one or more biopsies, in the following year than those who have a negative test result.

There is also convincing evidence that a substantial percentage of men who have asymptomatic cancer detected by PSA screening have a tumor that either will not progress or will progress so slowly that it would have remained asymptomatic for the man’s lifetime.

Although the precise, long-term effect of PSA screening on prostate cancer-specific mortality remains uncertain, existing studies adequately demonstrate that the reduction in prostate cancer mortality after 10 to 14 years is, at most, very small, even for men in what seems to be the optimal age range of 55 to 69 years. There is no apparent reduction in all-cause mortality. In contrast, the harms associated with the diagnosis and treatment of screen-detected cancer are common, occur early, often persist, and include a small but real risk of premature death.

Additional Reading: U.S. Preventive Services Task Force. Screening for prostate cancer: Recommendation statement. Am Fam Physician. 2013;87(4):267-273.

Category: Reproductive system



23. Type II renal tubular acidosis (RTA) is associated with


A) The proximal tubules having decreased ability to absorb bicarbonate


B) Urine pH that is normal when plasma bicarbonate levels are normal


C) Chronic metabolic alkalosis


D) Plasma bicarbonate levels that are easily restored with supplementation


E) Hyperkalemia

View Answer

Answer and Discussion

The answer is A. Type I (distal) RTA is a disorder that affects adults and is considered a familial disorder in children. Sporadic cases may be primary (especially in women) or secondary (e.g., to an autoimmune disease such as Sjögren’s syndrome; medications including amphotericin B or lithium therapy; kidney transplantation; nephrocalcinosis; renal medullary sponge kidney; chronic renal obstruction). Familial cases may be autosomal dominant and are often associated with hypercalciuria. In type I RTA, the urine pH is never <5.5.

Type II (proximal) RTA is associated with several inherited diseases (e.g., Fanconi’s syndrome, fructose intolerance, Wilson’s disease, Lowe’s syndrome), multiple myeloma, vitamin D deficiency, and chronic hypocalcemia with secondary hyperparathyroidism. It may occur after renal transplant, exposure to heavy metals, and after treatment with certain medications, including acetazolamide, sulfonamides, tetracycline, and streptozocin. In type II RTA, the ability of the proximal tubules to reabsorb HCO3 is decreased, so that urine pH is >7 at normal levels of plasma HCO3, but may be <5.5 at low levels of plasma HCO3. Type III RTA is a combination of types I and II and is seldom seen.

Type IV RTA is a condition associated with mild renal insufficiency in adults with diabetes mellitus, human immunodeficiency virus (HIV) nephropathy, or interstitial renal damage (SLE, obstructive uropathy, sickle cell disease). It may also be produced by drugs that interfere with the renin-aldosterone system (e.g., NSAIDs, angiotensin-converting enzyme [ACE] inhibitors, potassium-sparing diuretics, trimethoprim). Aldosterone deficiency or unresponsiveness of the distal tubule to aldosterone results in type IV RTA. This reduces potassium excretion, causing hyperkalemia, which reduces ammonia production and acid excretion by the kidney. Urine pH is usually normal.

Types I and II RTA are associated with chronic metabolic acidosis, mild volume loss, and hypokalemia. Hypokalemia may lead to muscle weakness, hyporeflexia, and paralysis. Type I RTA has decreased citrate excretion in the urine, increased mobilization of bone calcium, and hypercalciuria, which results in osteopenia, bone pain, and kidney stones or nephrocalcinosis. Renal parenchymal damage and chronic renal failure may develop. Type IV RTA is usually asymptomatic with only mild acidosis, but cardiac arrhythmias or paralysis may develop if hyperkalemia is extreme. Sodium bicarbonate relieves symptoms and prevents or stabilizes renal failure and bone disease. In adults with type I RTA, sodium bicarbonate eliminates acidosis and reduces the occurrence of kidney stones. In type II RTA, the plasma HCO3 cannot be restored to the normal range. HCO3 replacement should exceed the acid load of the diet. Additional HCO3 replacement increases potassium bicarbonate losses in the urine. Citric acid/sodium citrate (Bicitra) or potassium citrate/citric acid (Polycitra-K) can be substituted for sodium bicarbonate and may be better tolerated. Potassium supplements may be required in patients who become hypokalemic when given sodium bicarbonate, but are not recommended in patients with normal or high serum potassium levels. In type IV RTA, the hyperkalemia is treated with fluid administration and potassium-depleting diuretics. A few patients may need mineralocorticoid replacement therapy.

Additional Reading: Overview of renal tubular acidosis. In: Basow DS, ed. UpToDate. Waltham, MA: UpToDate; 2013.

Category: Nephrologic



24. Which of the following is considered a first-line medication in the treatment of hypertension?


A) Chlorthalidone


B) Lisinopril


C) Clonidine


D) Losartan


E) Amlodipine


View Answer

Answer and Discussion

The answer is A. A 2009 Cochrane review was undertaken to compare the effects of thiazide diuretics, β-blockers, ACE inhibitors, and CCBs with placebo. Thiazide diuretics (e.g., chlorthalidone, hydrochlorothiazide) lowered mortality and morbidity from stroke, heart attack, and heart failure more than β-blockers. ACE inhibitors (e.g., lisinopril) and CCBs (e.g., amlodipine) reduced mortality and morbidity as much as thiazide diuretics, but the evidence is less robust. Because the use of thiazide diuretics is supported by a strong body of evidence and no other class of antihypertensive medications has been shown to be better at improving outcomes, they are the firstline drugs for most patients with hypertension.

Doxazosin is an α-blocker and is not indicated for first-line treatment. Losartan is an angiotensin-receptor blocker (ARB), which should be reserved for patients who cannot tolerate ACE inhibitors. Unfortunately, there are no high-quality randomized controlled trials (RCTs) to fully evaluate the effects of ARBs or α-blockers and current evidence does not support using β-blockers as first-line therapy for hypertension.

Diuretics remain the preferred first-step drug and an important part of any multidrug regimen for the treatment of hypertension. ACE inhibitors and CCBs appear to be as effective as thiazide diuretics, but the evidence is not as strong.

Additional Reading: First-line treatment for hypertension. Am Fam Physician. 2010;81(11):1333-1335.

Category: Cardiovascular system




25. Which of the following drugs offers protection from osteoporosis?


A) Hydrochlorothiazide


B) Metoprolol


C) Enalapril


D) Verapamil


E) Losartan

View Answer

Answer and Discussion

The answer is A. In healthy elderly adults, low-dose hydrochlorothiazide preserves bone mineral density at the hip and spine. Hydrochlorothiazide produces small positive benefits on cortical bone density that are sustained for at least the first 4 years of treatment. They provide a further option in the prevention of postmenopausal bone loss, especially for women with hypertension or a history of kidney stones. None of the other listed medications provides this protection.

Additional Reading: The effect of treatment with a thiazide diuretic for 4 years on bone density in normal postmenopausal women. Osteoporo Int. 2007;18(4):479-486.

Category: Endocrine system



26. An ankle-brachial index of _______ is considered normal.


A) 0.95


B) 0.75


C) 0.50


D) 0.25


E) 0.15

View Answer

Answer and Discussion

The answer is A. Symptoms of claudication include pain, ache, cramp, or tired feeling that occurs on walking. They are most common in the calf but may occur in the foot, thigh, hip, or buttocks. The condition is worsened by walking rapidly or uphill and usually relieved in 1 to 5 minutes by rest (sitting is not necessary); the patient can walk the same distance again before pain recurs. Disease progression is indicated by a reduction in the distance that the patient can walk without symptoms. Eventually, ischemic pain may occur at rest, beginning in the most distal parts of a limb as a severe, unrelenting pain aggravated by elevation and often interfering with sleep. If intermittent claudication is the only symptom, the extremity may appear normal, but the pulses are reduced or absent. The level of arterial occlusion and the location of intermittent claudication closely correlate (e.g., aortoiliac disease frequently causes claudication in the buttocks, hips, and calves, and the femoral pulses are reduced or absent). In men, impotence is common and depends on the location and extent of occlusion. In femoropopliteal disease, claudication is typically in the calf, and all pulses below the femoral are absent. In patients with small vessel disease (e.g., thromboangiitis obliterans, diabetes mellitus), femoropopliteal pulses may be present, but foot pulses are absent. Pallor of the involved foot after 1 to 2 minutes of elevation, followed by redness on dependency, helps confirm arterial insufficiency. Normal venous filling time with dependency after elevation is 15 seconds. If symptoms of claudication occur with good distal pulses, spinal stenosis should be considered. A severely ischemic foot is painful, cold, and often numb. In chronic cases, the skin may be dry and scaly, with poor nail and hair growth. As ischemia worsens, ulceration may appear (typically on the toes or heel, occasionally on the leg), especially after local trauma. Edema is usually not present unless the patient has kept the leg in a dependent position for pain relief. More extensive blockage may compromise tissue viability, leading to necrosis or gangrene. Ischemia with redness, pain, and swelling of the foot on dependency may mimic cellulitis or venous insufficiency. Although arterial occlusion in the extremities can usually be diagnosed clinically, noninvasive tests confirm the diagnosis and are useful in follow-up. Invasive tests can document the location and extent of disease if angioplasty, local fibrinolytic therapy, or surgical bypass is contemplated. Doppler ultrasonography is most widely used. Arterial stenosis and occlusion can be detected using a velocity detector (Doppler probe). A colored signal shows the direction of flow (color Doppler). The simplest method for estimating blood flow to the lower extremities is to compare systolic BP at the level of the ankle with brachial systolic pressure (ankle-brachial indices). During this procedure, a blood pressure cuff is applied to the ankle, inflated above brachial systolic pressure, and deflated slowly. Ankle systolic blood pressure can be obtained accurately with a Doppler probe placed over the dorsalis pedis or posterior tibial arteries. This blood pressure at rest normally is ≥90% of the brachial systolic pressure; with mild arterial insufficiency, it is 70% to 90%; with moderate insufficiency, 50% to 70%; and with severe insufficiency, <50%.

Additional Reading: Critical review of the ankle brachial index. Curr Cardiol Rev. 2008;4(2):101-106.

Category: Cardiovascular system



27. A secondary cause of RLS is


A) Vitamin B12 deficiency


B) Heavy metal intoxication


C) Alcohol abuse


D) Iron deficiency


E) Bismuth overdose


View Answer

Answer and Discussion

The answer is D. RLS is a neurologic movement disorder that is often associated with a sleep disturbance. Patients with RLS have an irresistible urge to move their legs, which is usually secondary to uncomfortable sensations that are worse during periods of inactivity and often interfere with sleep. It is estimated that between 2% and 15% of the population may experience symptoms of RLS. Primary RLS may have a genetic origin. The diagnosis of RLS is based primarily on the patient’s history. Pharmacologic treatment of RLS includes dopaminergic agents, opioids, benzodiazepines, and anticonvulsants.

Secondary causes of RLS include iron deficiency, neurologic lesions, pregnancy, and uremia. Patients with iron deficiency may receive symptom relief by taking supplemental iron. A ferritin level of less than 50 ng per mL may cause or exacerbate RLS. Although levels above 10 to 20 ng per mL are reported as normal, supplemental iron may improve symptoms in individuals with levels less than 50 ng per mL. Iron is not beneficial in individuals with ferritin above this level. RLS also may occur secondarily to the use of certain medications.

Additional Reading: Restless legs syndrome. Am Fam Physician. 2008;78(2):235-240.

Category: Neurologic



28. In appropriate patients, thrombolytic therapy can be given up to __hours after onset of stroke symptoms.


A) 1 hour


B) 2 hours


C) 3 hours


D) 24 hours


E) 36 hours

View Answer

Answer and Discussion

The answer is C. Transient ischemic attack (TIA) is considered a significant warning sign of impending stroke. It is crucial to recognize these events to prevent permanent disability or death in affected individuals. The 90-day risk of stroke after a TIA has been estimated to be approximately 10%, with one-half of strokes occurring within the first 2 days of the attack. The 90-day stroke risk is even higher when a TIA results from internal carotid artery disease. Most patients reporting symptoms of TIA should be referred to an emergency department for further evaluation. Patients who arrive at the emergency department within 180 minutes of symptom onset should undergo evaluation to determine if they are candidates for thrombolytic therapy. Initial testing should include complete blood count with platelet count, prothrombin time, international normalized ratio, partial thromboplastin time, and electrolyte and glucose levels. CT scanning of the head should be performed immediately to ensure that there is no evidence of brain hemorrhage or mass. Risk factors for stroke should be evaluated in patients who have had a TIA. Blood pressure, lipid levels, and diabetes mellitus should be controlled. If indicated, smoking cessation and weight loss are also important. ACE inhibitor therapy may help prevent stroke. Aspirin is the treatment of choice for stroke prevention in patients who do not require anticoagulation. Clopidogrel (Plavix) is an alternative therapy in patients who do not tolerate aspirin. Atrial fibrillation, a known cardioembolic source (confirmed thrombus), or a highly suspected cardioembolic source (e.g., recent large MI, dilated cardiomyopathy, mechanical valve, rheumatic mitral valve stenosis) is an indication for anticoagulation with warfarin therapy.

Additional Reading: Transient ischemic attacks: Part I. Diagnosis and evaluation. Am Fam Physician. 2004;69:1665-1674, 1679-1680. Transient ischemic attacks: Part II. Treatment. Am Fam Physician. 2004;69:1681-1688.

Category: Neurologic



29. Of the following, which is least likely to be seen in strep throat?


A) Fever


B) Malaise


C) Tonsilar exudates


D) Palatine petechiae


E) Rhinorrhea

View Answer

Answer and Discussion

The answer is E. Sore throat is one of the most common reasons for visits to family physicians. Although most patients with sore throat have an infectious cause (pharyngitis), <20% have a clear indication for antibiotic therapy (i.e., group A β-hemolytic streptococcal infection). Viral pharyngitis is the most common cause of sore throat. Infectious mononucleosis (IM) is most common in patients 15 to 30 years of age. Patients typically present with fever, sore throat, and malaise. On examination, there is pharyngeal redness with exudates. Posterior cervical lymphadenopathy is common in patients with IM, and its absence makes the diagnosis much less likely. Hepatosplenomegaly also may be present. If these patients are treated with amoxicillin or ampicillin, 90% develop a classic maculopapular rash. Patients with bacterial pharyngitis generally do not have rhinorrhea, cough, or conjunctivitis. Children younger than 15 are more likely to have strep throat. Symptoms of strep throat may include pharyngeal erythema and swelling, tonsillar exudate, edematous uvula, palatine petechiae, and anterior cervical lymphadenopathy. Untreated, strep pharyngitis lasts 7 to 10 days. Patients with untreated streptococcal pharyngitis are infectious during the acute phase of the illness and for 1 additional week. Antibiotic therapy shortens the infectious period to 24 hours, reduces the duration of symptoms by about 1 day, and prevents most complications. The incidence of complications with strep infection, such as rheumatic fever and peritonsillar abscess, is low. Peritonsillar abscess occurs in <1% of patients treated with antibiotics. Patients with peritonsillar abscess typically have a toxic appearance and may present with a muffled voice, fluctuant peritonsillar mass, and asymmetric deviation.

Additional Reading: Pharyngitis. Am Fam Physician. 2004;69: 1465-1470.

Category: Respiratory system



30. Angioneurotic edema is associated with the use of


A) ACE inhibitors


B) β-Blockers


C) Loop diuretics


D) α-Receptor blockers


E) CCBs

View Answer

Answer and Discussion

The answer is A. Angioneurotic edema, which occurs in 0.1% to 0.2% of patients, usually develops within the first week of therapy but can occur at any time. This life-threatening adverse effect also occurs with angiotensin II-receptor blockers, but to a lesser extent. Any patient with a history of angioneurotic edema, whether related to an ACE inhibitor, ARBs, or another cause, should not be given an ACE inhibitor. Other contraindications include pregnancy, renal artery stenosis, and previous allergy to ACE inhibitors.


Additional Reading: Drugs for Hypertension. Treat Guidel Med Lett. 2012;10(113):1-10.

Category: Cardiovascular system



31. Genital warts


A) Rarely resolve spontaneously


B) Are treated on the basis of cost, convenience, and adverse affects


C) Do not remain in tissue after treatment


D) Are treated with an alternative method if a single treatment fails to eradicate the wart

View Answer

Answer and Discussion

The answer is B. Untreated visible genital warts may resolve spontaneously, remain the same, or increase in size. The primary treatment goal is removal of symptomatic warts. Some evidence suggests that treatment may also reduce the persistence of human papillomavirus (HPV) DNA in genital tissue and therefore may reduce the incidence of cervical cancer. The choice of therapy is based on the number, size, site, and morphology of lesions, as well as patient preference, treatment cost, convenience, adverse effects, and physician experience. Assuming that the diagnosis is certain, switching to a new treatment modality is appropriate if there is no response after three treatment cycles. Routine follow-up at 2 to 3 months is advised to monitor response to therapy and evaluate for recurrence. Treatment methods can be chemical or ablative.

Additional Reading: Management of genital warts. Am Fam Physician. 2004;70:2335-2342, 2345-2346.

Category: Integumentary



32. In which of the following cases is comprehensive evaluation necessary when a patient is affected by a deep venous thrombosis (DVT)?


A) 45-year-old man with an idiopathic DVT


B) 65-year-old with a recent transatlantic flight and DVT of left thigh


C) 55-year-old who develops a calf DVT after a 4-hour car ride


D) 75-year-old with a history of non-small cell cancer of lung with left leg DVT


E) 72-year-old with a right thigh DVT and no history of travel

View Answer

Answer and Discussion

The answer is A. Treatment goals for DVT include stopping clot propagation and preventing the recurrence of thrombus, the occurrence of PE, and the development of pulmonary hypertension, which can be a complication of multiple recurrent pulmonary emboli. About 30% of patients with DVT or PE have a thrombophilia. A comprehensive evaluation is suggested in patients younger than 50 years with an idiopathic episode of DVT, patients with recurrent thrombosis, and patients with a family history of thromboembolism. Intravenous administration of unfractionated heparin followed by oral administration of warfarin remains the mainstay of treatment for DVT. Subcutaneous low-molecular-weight (LMW) heparin is at least as effective as unfractionated heparin given in a continuous intravenous route. LMW heparin is the agent of choice for treating DVT in pregnant women and patients with cancer. On the basis of validated protocols, warfarin can be started at a dosage of 5 or 10 mg per day. The intensity and duration of warfarin therapy depends on the individual patient, but treatment of at least 3 months usually is required. Some patients with thrombophilias require lifetime anticoagulation. Treatment for PE is similar to that for DVT. Because of the risk of respiratory failure and hemodynamic instability, in-hospital management is advised. Unfractionated heparin is commonly used, although LMW heparin is safe and effective. Thrombolysis is used in patients with massive PE. Subcutaneous heparin, LMW heparin, and warfarin have been approved for use in surgical prophylaxis. Elastic compression stockings are useful in patients at lowest risk for thromboembolism. Intermittent pneumatic leg compression is a useful adjunct to anticoagulation and an alternative when anticoagulation is contraindicated.

Additional Reading: Diagnosis of deep venous thrombosis and pulmonary embolism. Am Fam Physician. 2012;86(10):913-919

Category: Cardiovascular system



33. Which of the following statements is correct concerning hepatitis C virus (HCV)?


A) There is no risk to infants if the mother is affected.


B) There is no risk associated with sexual intercourse with an individual with hepatitis C.


C) Cesarean section should be performed on mothers who test positive for hepatitis C to prevent transmission to the newborn.


D) Hepatitis C can be spread by contaminated water supplies.


E) Hepatitis C does not appear to be transmitted in breast milk.

View Answer

Answer and Discussion

The answer is E. In an effort to reduce the risk of transmission to others, HCV-positive patients should be advised not to donate blood, organs, tissue, or semen; not to share toothbrushes, dental appliances, razors, or other personal care articles that might have blood on them; and to cover cuts and sores on the skin to keep from spreading infectious blood or secretions. HCV-positive patients with one long-term, steady sex partner do not need to change their sexual practices. They should, however, discuss the risk (which is low but not absent) with their partner. If they want to lower the small chance of spreading HCV to their partner, they may decide to use barrier precautions such as latex condoms. HCV-positive women do not need to avoid pregnancy or breast-feeding. Potential, expectant, and new parents should be advised that about 5 of every 100 infants born to HCV-infected women become infected. This infection occurs at the time of birth, and no treatment has been shown to prevent the transmission. There is no evidence that the method of delivery is related to transmission; therefore, the need for cesarean section versus vaginal delivery should not be determined on the basis of HCV-infection status. Limited data on breast-feeding indicate that it does not transmit HCV, although it may be prudent for HCV-positive mothers to abstain from breast-feeding if their nipples are cracked or bleeding. Infants born to HCV-positive women should be tested for HCV infection and, if positive, evaluated for the presence or development of chronic liver disease. HCV is not spread by sneezing, hugging, coughing, food or water, sharing eating utensils or drinking glasses, or casual contact. Persons should not be excluded from work, school, play, child care, or other settings on the basis of HCV-infection status. HCV-positive persons should be evaluated to assess for biochemical evidence of chronic liver disease. These patients should be assessed for severity of disease and possible treatment according to current practice guidelines in consultation with, or by referral to, a specialist knowledgeable in this field.

Additional Reading: Hepatitis C: diagnosis and treatment. Am Fam Physician. 2010;81(11):1351-1357.

Category: Gastroenterology system





34. Which of the following classes of drugs can be used safely with NSAIDs without needing to closely monitor the patient’s renal function, potassium levels, and/or blood pressure?


A) ACE inhibitors


B) CCBs


C) Diuretics


D) β-Blockers

View Answer

Answer and Discussion

The answer is B. Combined use of NSAIDs and hypertensive medication (i.e., diuretics, β-blockers, α-blockers, and ACE inhibitors) may decrease the effectiveness of antihypertensive medication and cause serious complications. Thus, when using the two in combination, renal function, potassium levels, and blood pressure should be monitored. CCBs and central α-agonists can usually be used without these concerns.

Additional Reading: Mechanisms and treatment of resistant hypertension. J Clin Hypertens (Greenwich). 2008;10(3):239-244.

Category: Cardiovascular system



35. Which of the following statements about sunscreens and sun exposure is true?


A) The most dangerous rays are the ultraviolet A (UVA) type.


B) Sunscreens with a skin protection factor of 10 are adequate protection.


C) Patients allergic to thiazide diuretics may react adversely to para-aminobenzoic acid (PABA).


D) Steroids should be avoided in patients with sunburns because of their immunosuppressant properties.


E) Repeated use of sunscreens can increase the risk of sun poisoning.

View Answer

Answer and Discussion

The answer is C. Sun-produced UV light is divided into two types of rays: UVA (320 to 400 nm) and UVB (280 to 320 nm). The dangerous rays are in the UVB range. Sunscreens of at least skin protection factor 15 (and preferably SPF-30) should be used when persons are exposed to the sun. Para-aminobenzoic acid, which is used in many sunscreens, is very effective at preventing sunburns. Unfortunately, patients with sensitivities to thiazides, benzocaine, or sulfonamides may react adversely to para-aminobenzoic acid. In most cases, sunburn is prevented with simple precautions. Sunburn (usually a first-degree burn) appears within the first 24 hours and can be very painful. Sunburn is treated with cold-water compresses. In severe cases, sunburns can be treated with steroids.

Additional Reading: Sunscreens revisited. Med Lett Drugs Ther. 2011;53(1359):17-18.

Category: Integumentary



36. Which of the following tests is recommended to be used to screen when celiac sprue is suspected?


A) Shilling’s test


B) Antigliadin IgA and IgG antibodies


C) IgA tissue transglutaminase antibodies (tTG)


D) Withdrawal of lactose from the diet to monitor for improvement of symptoms


E) Scotch tape test

View Answer

Answer and Discussion

The answer is C. Celiac sprue is an inherited disorder that is characterized by an intolerance to gluten, a cereal-type protein found in wheat, rye, oats, and barley. Symptoms in infancy include colic, failure to thrive, and, in severe cases, iron-deficiency anemia with the development of edema. In adults, symptoms include abdominal bloating and discomfort, with diarrhea, anemia, weight loss, arthralgias, and edema. Steatorrhea is usually present. Laboratory findings usually include iron-deficiency anemia (in children), folate-deficiency anemia (in adults), low protein levels, and electrolyte abnormalities and coagulation studies may be abnormal.

Celiac disease can be difficult to diagnose and may be confused with irritable bowel syndrome, inflammatory bowel disease, diverticulitis, intestinal infections, iron-deficiency anemia caused by menstrual blood loss, and chronic fatigue syndrome. IgA tissue transglutaminase antibodies (tTG) and IgA endomysial antibodies (EMA) are appropriate first-line serologic tests to rule in celiac disease. The tTG test uses a less costly enzyme-linked immunosorbent assay (ELISA); therefore, it is the recommended single serologic test for celiac disease screening in the primary care setting. However, a minority of patients with celiac disease have IgA deficiency. Therefore, if the serum IgA tTG result is negative but clinical suspicion for the disease is high, a serum total IgA level may be considered. Antigliadin IgA and IgG antibodies are elevated in >90% of patients; however, they are nonspecific and no longer recommended to test for celiac disease. A positive IgA tTG result should prompt small bowel biopsy to confirm the diagnosis. A jejunum biopsy would show a flat mucosa with a loss of intestinal villi. Before being tested, one should continue to eat a diet that includes foods with gluten, such as breads and pastas. If a person stops eating foods with gluten before being tested, the results may be negative for celiac disease even if the disease is present.

The Schilling test is used to diagnose pernicipous aneamia in patients with vitamin B12 deficiency, and the Scotch tape test is used in the diagnosis of pin worms. Withdrawal of lactose from the diet to monitor for improvement of symptoms can be utilized when assessing patients for lactose deficiency.

Treatment involves dietary counseling to avoid gluten-containing foods and supplementary vitamins. In severe cases, corticosteroids are used to induce a refractory stage.

Additional Reading: Celiac disease. Am Fam Physician. 2007; 76(12):1795-1802.

Category: Gastroenterology system



37. Pain associated with the distal second metatarsal head is most likely a result of


A) Morton’s neuroma


B) Jones fracture


C) Gout


D) Metatarsalgia

View Answer

Answer and Discussion

The answer is D. Metatarsalgia is characterized by pain and sometimes swelling associated with the second (and, less commonly, the third) metatarsal head. The pain is secondary to a synovitis that affects the joint. Patients with hammertoes are at an increased risk because of stress placed at the head of the metatarsals. In most cases, radiographs are normal; however, more severe cases may show subluxation or dislocation of the metacarpal joint. NSAIDs, hot soaks, and metatarsal pads may help; however, if subluxation or dislocation is present, surgery may be necessary. Morton’s neuroma usually presents with a feeling as if the patient is standing on a pebble in their shoe, with a burning sensation in the ball of your foot that may radiate into the toes. A Jones fracture is a fairly common fracture of the fifth metatarsal, and gout typically involves pain and inflammation at the first metatarsophalangeal joint, although other toes can be involved.


Additional Reading: Busconi BD, Stevenson JH. Approach to the athlete with metatarsalgia. Sports Medicine Consult: A Problem-Based Approach to Sports Medicine for the Primary Care Physician. Philadelphia, PA: Lippincott, Williams & Wilkins; 2009

Category: Musculoskeletal system



38. Desensitization immunotherapy may be used in the treatment of


A) Chronic urticaria


B) Hymenoptera allergies


C) Atopic dermatitis


D) Milk allergy


E) None of the above

View Answer

Answer and Discussion

The answer is B. Desensitization immunotherapy is used in the treatment of severe allergic rhinitis and beesting (hymenoptera) allergies. The patient is given gradually increasing concentrations of the allergen over an increasing period. Typically, there is a decrease in the mast-cell response with a decrease in histamine production when the patient is exposed to the allergen. In addition, IgE levels decrease. In most cases, the injections are continued year-round and may be spaced out as the desired response occurs. Injections should always be given in the presence of a physician, and appropriate equipment must be available to treat potential anaphylaxis. Patients must be observed for at least 30 minutes after administration of the injections. Desensitization immunotherapy is not appropriate for the treatment of chronic urticaria, milk allergies, or atopic dermatitis.

Additional Reading: Allergen immunotherapy. Am Fam Physician. 2004;70(4):689-696.

Category: Nonspecific system



39. A 21-year-old man is brought to the emergency room after suffering a seizure. He is tachycardic and hypertensive and has a temperature of 38.6°C. Physical examination shows that the patient is in a postictal state and has a nasal septum perforation. Electrocardiogram (ECG) findings suggest acute MI. Friends of the patient report recent cocaine use. Which of the following drugs is indicated?


A) Elumazenil


B) Diazepam


C) Dexfenfluramine


D) Propranolol


E) Phenytoin

View Answer

Answer and Discussion

The answer is B. Cocaine is a strong narcotic stimulant that is often abused. Its mechanism of action involves the increased release of norepinephrine and the blockage of its reuptake. Effects of cocaine begin within 3 to 5 minutes (within 8 to 10 seconds with smoking “crack” cocaine), and peak effects occur at 10 to 20 minutes. The effects rarely last more than 1 hour. Cocaine toxicity is characterized by seizures; hyperpyrexia; tachycardia; mental status changes, including paranoid behavior; hypertension; cerebrovascular accidents; MIs; and rhabdomyolysis. Nasal septum perforation may also occur.

In most cases, treatment involves the use of diazepam for aggitation, hypertension, and tachycardia. Nitroprusside or CCBs can be used for hypertensive crisis. β-Blockers should be avoided given the risk of coronary vasoconstriction and paradoxical hypertension. Other complications, such as cerebrovascular accidents, rhabdomyolysis, and MIs, should be managed in a conventional manner. Fortunately, cocaine has a short half-life and symptoms are usually self-limited. Individuals who use cocaine may become rapidly addicted.

Additional Reading: Cocaine overdose. Epocrates online. From: https://online.epocrates.com/

Category: Psychogenic



40. Goodpasture’s syndrome is associated with


A) Osteoporosis and renal lithiasis


B) Pathologic fractures and thyroiditis


C) Hepatitis and recurrent cystitis


D) Pulmonary hemorrhage and glomerulonephritis


E) Pica and angioedema

View Answer

Answer and Discussion

The answer is D. Goodpasture’s syndrome is a condition manifested by pulmonary hemorrhages and progressive glomerulonephritis. Circulating basement membrane antibodies are responsible for the renal and pulmonary abnormalities. Patients with Goodpasture’s syndrome are typically young males (5 to 40 years; male:female ratio of 6:1); however, there is a bimodal peak at approximately 60 years of age. Men and women are equally affected at older ages. Symptoms include severe hemoptysis, shortness of breath, and renal failure. Laboratory findings include iron-deficiency anemia, hematuria, proteinuria, cellular and granular casts in the urine, and circulating antiglomerular antibodies. Chest radiographs show progressive, bilateral, fluffy infiltrates that may migrate and are asymmetrical. Renal biopsy may be necessary to make the diagnosis. Treatment involves high-dose steroids, immunosuppression, and plasmapheresis, which may help preserve renal function. If significant injury to the kidneys occurs, then dialysis or transplant may be necessary. Untreated, Goodpasture’s syndrome can be fatal.

Additional Reading: Pathogenesis and diagnosis of anti-GBM antibody (goodpasture’s) disease. In: Basow DS, ed. UpToDate. Waltham, MA: UpToDate; 2013.

Category: Nephrologic system




41. Black cohosh has been advocated to treat


A) Muscle and joint pain


B) The common cold


C) Depression


D) Menopausal symptoms


E) Osteoporosis

View Answer

Answer and Discussion

The answer is D. The herb black cohosh, or Actaea racemosa (formerly named Cimicifuga racemosa), is native to North America. The roots and rhizomes of this herb are widely used in the treatment of menopausal symptoms and menstrual dysfunction. Although the clinical trials on black cohosh are of insufficient quality to support definitive statements, this herbal medicine may be effective in the short-term treatment of menopausal symptoms. The mechanism of action is unclear, and early reports of an estrogenic effect have not been proved in recent studies. Although black cohosh may be useful in treating some menopausal symptoms, there is currently no evidence regarding any protective effect of black cohosh against the development of osteoporosis. Adverse effects are extremely uncommon, and there are no known significant adverse drug interactions.


Additional Reading: Black cohosh. Am Fam Physician. 2003; 68:114-116.

Category: Nonspecific system



42. A 32-year-old man with a history of Wolff-Parkinson-White syndrome (WPW) presents to the emergency department with his “heart racing again.” He is alert and in no acute distress. His blood pressure is 130/70 mm Hg, pulse rate 220 beats per minute, and oxygen saturation 96%. An EKG reveals a regular, wide-complex tachycardia. He appears stable and his EKG is consistent with WPW. You decide to use pharmacologic conversion for his initial treatment; which one of the following would be the treatment of choice?


A) Verapamil (Calan)


B) Adenosine (Adenocard)


C) Procainamide


D) Digoxin

View Answer

Answer and Discussion

The answer is C. Adenosine, digoxin, and calcium-channel antagonists act by blocking conduction through the atrioventricular (AV) node, which may increase the ventricular rate paradoxically, initiating ventricular fibrillation. These agents should be avoided in Wolff-Parkinson-White syndrome. Procainamide is usually the treatment of choice in these situations, although amiodarone may also be used.

Additional Reading: Evaluation and initial treatment of supraventricular tachycardia. N Engl J Med. 2012;367:1438-1448.

Category: Cardiovascular system



43. A recognized complication of sleep apnea is


A) Hyperlipidemia


B) Diabetes mellitus


C) RLS


D) Migraines


E) Congestive heart failure (CHF)

View Answer

Answer and Discussion

The answer is E. Obstructive sleep apnea occurs most often in moderately or severely obese persons. Men are affected more often than women (4% of men and 2% of women in middle age). Upper airway narrowing leads to obstruction during sleep. In severely obese persons, a combination of hypoxemia and hypercapnia may induce central apnea as well. By definition, apneic periods last at least 10 seconds (some for 2 minutes). Repeated nocturnal obstruction may cause recurring cycles of sleep, obstructive choking, and arousal with gasping for air. Daytime drowsiness usually results from the repeated cycles. Similar but less-pronounced cycles occur in nonobese persons, possibly secondary to developmental or congenital abnormalities of the upper airway. Complications of sleep apnea include cardiac abnormalities (e.g., sinus arrhythmias, extreme bradycardia, atrial flutter, ventricular tachycardia, heart failure), hypertension, excessive daytime sleepiness, morning headache, and slowed mentation. The mortality rate from stroke and MIs is significantly higher in persons with obstructive sleep apnea than in the general population.

Additional Reading: Sleep apnea, obstructive. In: Domino F, ed. The 5-Minute Clinical Consult. Philadelphia, PA: Lippincott Williams and Wilkins; 2014.

Category: Neurologic



44. Which of the following medications is effective for RLS?


A) Levodopa/carbidopa


B) Diltiazem


C) Phenytoin


D) Haloperidol


E) Vitamin B6

View Answer

Answer and Discussion

The answer is A. RLS is a relatively common problem seen by family physicians. The condition is characterized by repeated movements and paresthesias of the lower extremities (occasionally the arms). Patients may describe a tingling irritation or a drawing or crawling sensation that prevents the onset of sleep or may disturb sleep. The symptoms are often relieved by movement. Laboratory and neurologic tests are normal. Associated conditions include iron deficiency, diabetes, uremia, pregnancy, rheumatoid arthritis, vitamin B12 deficiency, and polyneuropathy. Treatment includes the use of ropinirole (Requip), pramipexole (Mirapex), pergolide (Permax), levodopa/carbidopa (Sinemet), gabapentin (Neurontin), carbamazepine (Carbatrol), and other antiepileptics, opiates, and benzodiazepines.

Additional Reading: Restless legs syndrome. Am Fam Physician. 2008;78(2):235-240.

Category: Neurologic



45. Which of the following forms of hepatitis does NOT have a chronic state?


A) Hepatitis A


B) Hepatitis B


C) Hepatitis C


D) Hepatitis D

View Answer

Answer and Discussion

The answer is A. Hepatitis is an inflammation of the liver that is characterized by nausea, anorexia, fever, right-upper abdominal discomfort, jaundice, and marked elevation of liver function tests. The condition is usually classified into the following types:



  • Hepatitis A. Also known as infectious hepatitis, the causative agent is an RNA virus. The disease is common and often presents subclinically. It is estimated that as much as 75% of the U.S. population has positive antibodies to hepatitis A. The onset of clinical symptoms is usually acute, and children and young adults are usually affected. The transmission is via a fecal-oral route and has been linked to the consumption of contaminated shellfish (e.g., raw oysters). The course of the disease is usually mild, and the prognosis is usually excellent. There is neither an associated chronic state nor a carrier state. The diagnosis is made by the detection of elevated levels of IgM antibodies, which indicate active disease, and IgG antibodies, which indicate previous disease. Most cases require no special treatment other than supportive care, and symptoms usually resolve after several weeks. The disease can be prevented by administering Ig to those who are in close contact with those affected. Immunization, especially for travelers, is recommended to specifically prevent hepatitis A.


  • Hepatitis B. This DNA viral disease is more severe than hepatitis A and causes more complications. It affects as much as 10% of the U.S. population. The infective Dane particle consists of a viral core and outer surface coat. The disease often develops insidiously and can affect persons of all ages. It is transmitted parenterally (through infected blood transfusions or infected needles used by intravenous drug abusers) and through sexual contact (especially
    in sexually active young adults and homosexuals). The symptoms are often severe and can be devastating to elderly patients or those who are debilitated. Approximately 10% of cases become chronic; up to 30% of affected patients become carriers of the virus after they are infected. The detection of the hepatitis B surface antigen (formerly known as the Australian antigen) supports the diagnosis of acute illness, and values become positive between 1 and 7 weeks before the symptoms become evident. The hepatitis B antibody appears weeks to months after the development of the clinical symptoms. The presence of a hepatitis B surface antibody indicates previous disease and represents immunity. Those who have received hepatitis B vaccination also have positive titers if they are immune. An anticore antibody (IgM) usually develops at the onset of the illness, and the IgG anticore antibody (which develops shortly after IgM appears) can be used as a marker for the disease during the “window period,” which occurs when the hepatitis B surface antigen disappears and before the hepatitis B surface antibodies appear. The hepatitis B e antigen is found in those who are hepatitis B surface antigen-positive; its presence is associated with greater infectivity and a greater chance of progression to the chronic state. The delta agent (hepatitis D) is a separate virus that may coexist with hepatitis B; it is usually associated with a more severe case of hepatitis B and in cases of chronic hepatitis B in which there is reactivation of the virus. Prophylaxis of hepatitis B can be achieved with hepatitis B vaccine given at 1 month and 6 months after the initial injection, for a total of three injections. Persons exposed to hepatitis B (e.g., by needle stick) should also receive hepatitis B Ig at the time of exposure.


  • Hepatitis C. This disease (also known as non-A, non-B hepatitis, or posttransfusion hepatitis) accounts for as many as 40% of the cases of hepatitis in the United States. It is the main indication for liver transplant in the United States when cirrhosis is present. The disease is transmitted by infected blood and is commonly seen in intravenous drug abusers and those who had blood transfusions infected with the virus. The disease is usually insidious in its presentation, and the severity is variable. As many as 50% of these patients may develop chronic disease, which may eventually lead to cirrhosis. The diagnosis is made by serologic means, and pegylated α-interferon and ribavirin have been used for treatment.


  • Hepatitis E. The transmission is similar to the hepatitis A virus. The disease is found in India and Southeast Asia, Africa, and Mexico. Cases in the United States are usually related to travel to these endemic areas. Hepatitis E virus is associated with a high fatality in pregnant women.

Additional Reading: Hepatitis A. Am Fam Physician. 2012;86(11): 1027-1034

Category: Gastroenterology system



46. Which of the following factors is included in the criteria for administering streptokinase with MI?


A) Cardiogenic chest pain lasting at least 6 hours


B) ECG changes of at least 1 to 2 mm of ST elevation in two adjacent precordial leads


C) Streptokinase should not be administered 6 hours after the onset of chest pain


D) Q waves noted in the lateral precordial leads

View Answer

Answer and Discussion

The answer is B. Streptokinase is a thrombolytic agent administered during MI. The majority of patients with an acute ST elevation myocardial infarction (STEMI) have a complete occlusion of a coronary artery due to thrombus. Thus, thrombolytic therapy has been important treatment option. However, primary percutaneous coronary intervention (PCI) is the preferred appraoch for reperfusion for most patients with STEMI as there are better outcomes (lower mortality and less recurrent ischemia) with fewer complications such as intracranial hemorrhage that occurs with thrombolytic use. However, such treatment remains an important reperfusion strategy in locations with limited availability of timely PCI.

Streptokinase is associated with a 1% to 2% rate of allergic reactions consisting of skin rashes and fever and ypotension occurs in another 10%. Because of the development of antibodies, patients previously treated with streptokinase should be given recombinant tissue plasminogen activator (alteplase) or reteplase plasminogen activator. The criteria for consideration of thrombolytics include chest pain (consistent with cardiogenic pain) for at least 30 minutes’ duration and ECG changes that show at least 1 to 2 mm of ST elevation in two adjacent precordial leads. Medication should be given within 12 hours for maximal benefit. Although extremely variable depending on the source, the following is a list of absolute contraindications to thrombolytics:



  • History of intracranial hemorrhage


  • Other strokes or cerebrovascular events within 1 year


  • Known intracranial neoplasm


  • Unclear mental status


  • Active gastrointestinal (GI) bleeding


  • Aortic dissection


  • Acute pericarditis

Relative contraindications include:



  • Recent surgery (within 3 weeks)


  • Prolonged (>10 minutes) cardiopulmonary resuscitation


  • Recent vascular puncture in a noncompressible region (<2 weeks)

Prior stroke (nonhemorrhagic)



  • Uncontrolled hypertension defined as systolic blood pressure (SBP) > 180 mm Hg or diastolic blood pressure (DBP) > 110 mm Hg

Major surgery (3 months or less)



  • Pregnancy


  • Bleeding diasthesis


  • Active peptic ulcer disease

Minor hemorrhage, menstruation, and diabetic retinopathy are not contraindications to fibrinolytic therapy.

Additional Reading: Fibrinolytic therapy in acute ST elevation myocardial infarction: initiation of therapy. In: Basow DS, ed. UpToDate. Waltham, MA: UpToDate; 2013.

Category: Cardiovascular system



47. A 48-year-old alcoholic stops drinking 2 days before presenting to his physician. He is diaphoretic, nauseated, tachycardic, anxious, and hypertensive. The most appropriate management is to


A) Prescribe diazepam and refer the patient to a drug treatment program


B) Hospitalize the patient, administer diazepam, and closely observe his condition


C) Administer disulfiram and diazepam and follow up with the patient in 1 week


D) Reassure the patient, compliment his decision to stop drinking, and explain the symptoms that are to be expected


E) Refer the patient to psychiatry


View Answer

Answer and Discussion

The answer is B. Symptoms of alcohol withdrawal usually occur 6 to 48 hours after the last alcoholic drink. These symptoms include sweating, anxiety, tremor, weakness, gastrointestinal (GI) discomfort, hypertension, tachycardia, fever, and hyperreflexia. Other symptoms include hallucinations and, in severe cases, delirium tremens that are characterized by disorientation with hallucinations, drenching sweats, severe tremors, and electrolyte disturbances that can lead to seizures. Treatment involves hospitalization and close observation. Antianxiety medications, including chlordiazepoxide, lorazepam, diazepam, midazolam, and oxazepam, are used for treatment and are slowly tapered to prevent withdrawal-related symptoms. Oral multivitamin supplementation with thiamine, folate, and pyridoxine is also recommended. Disulfiram is not used for alcohol withdrawal but can be used in the treatment of alcoholism to help discourage further drinking.

Additional Reading: Management of alcohol withdrawal syndrome. Am Fam Physician. 2010;82(4):344-347.

Category: Psychogenic



48. Which of the following statements about Burner’s syndrome is true?


A) The mechanism of injury involves acute hyperextension of the shoulder while the neck and head are forced in the same direction.


B) Symptoms include temporary weakness, pain, paresthesias, and decreased sensation of the distal extremity.


C) The injury involves traction forces on the spinal cord’s dorsal columns.


D) Most cases cause permanent neurologic deficits.


E) The condition is associated with overuse injury of the knees.

View Answer

Answer and Discussion

The answer is B. Burner’s syndrome is seen mostly in football players and results from a tackling or blocking injury. The injury occurs when the contact shoulder is depressed and the head and neck are forced in the opposite direction of contact. The tractiontype forces placed on the brachial plexus lead to variable symptoms of weakness, pain, paresthesias, limited motion, and decreased sensation of the affected extremity. Diminished reflexes may also be seen. The condition should be treated with caution, and cervical disc or bony injury should be ruled out. In most cases, the symptoms last only a few minutes; however, the athlete should not return to play until a complete evaluation can be performed and the symptoms resolve.

Additional Reading: Busconi BD, Stevenson JH. Approach to the athlete with burners/stingers (Transient brachial plexopathy). Sports Medicine Consult: A Problem-Based Approach to Sports Medicine for the Primary Care Physician. Philadelphia, PA: Lippincott, Williams & Wilkins; 2009.

Category: Musculoskeletal system




49. Which of the following indicates a therapeutic effect for β-blockers?


A) Pupillary constriction


B) Drug level within the acceptable range


C) Heart rate between 60 and 70 bpm


D) Generalized fatigue


E) Peripheral cyanosis

View Answer

Answer and Discussion

The answer is C. β-Blockers (e.g., propranolol, metoprolol, labetalol, nadolol) are used in the treatment of hypertension. They are considered a negative inotrope and chronotrope. In most cases, they are best suited for young patients who have a hyperdynamic cardiac status. β-Blockers should be used cautiously in patients with the following:



  • Asthma and COPD, because nonselective β-blockers can induce bronchoconstriction


  • Diabetes, because β-blockers can blunt the response of hypoglycemia


  • History of CHF, because β-blockers can decrease cardiac output (however, recent evidence supports ardioselective β-blocker use in CHF with systolic dysfunction)


  • Bradycardia or heart block

Other side effects include fatigue, impotence, impaired glucose tolerance, and rebound tachycardia and hypertension (if the drug is abruptly discontinued). β-Blockers are also used for migraine prophylaxis and to treat performance anxiety and tachycardia. Newer evidence supports that β-blockers are not deleterious for patients with depression as once thought. Finally, β-blockers are also used after MI to improve survival; they reduce myocardial oxygen demand by decreasing heart rate and contractility. Additionally, they should be given prior to surgery in those at risk for cardiac events. A therapeutic dose is determined by a recorded heart rate of 60 to 70 bpm.

Additional Reading: Drugs for hypertension. Treat Guidel Med Lett. 2012;10(113):1-10.

Category: Cardiovascular system



50. A 54-year-old woman presents to your office with complaints of frequent sweating episodes, palpitations, nervousness, and sensitivity to heat with increased appetite and weight loss. The most likely diagnosis is


A) Hypothyroidism


B) Menopause


C) Addison’s disease


D) Hyperthyroidism


E) Cushing’s disease

View Answer

Answer and Discussion

The answer is D. The manifestations of hyperthyroidism are numerous and include the following: goiter; widened pulse pressure; tachycardia; warm, moist skin; tremor; atrial fibrillation; nervousness; frequent diaphoresis; sensitivity to heat; palpitations; exophthalmos; pretibial myxedema; increased appetite with weight loss; diarrhea; and insomnia. The hallmark findings of Graves’ disease include the triad of goiter, exophthalmos, and pretibial myxedema. Anemia, present with hypothyroidism, is not seen with hyperthyroidism. While menopausal symptoms would include hot flashes and night sweats, mood changes are also often seen, but slowed metabolism and weight gain are more likely than weight lost and there would be accompanying irregular periods.


Addison’s disease is characterized by a gradual onset of fatigability, weakness, anorexia, nausea and vomiting, weight loss, skin and mucous membrane pigmentation, hypotension, and in some cases hypoglycemia depending on the duration and degree of adrenal insufficiency. The manifestations vary from mild chronic fatigue to life-threatening shock associated with acute destruction of the glands. Asthenia is the major presenting symptom.

Cushing’s disease is not associated with sweats and palpatations— weight gain, especially truncal obesity and “moon facies” are common. While depression and irritability can be seen, so can menstrual irregularities.

Additional Reading: Overview of the clinical manifestations of hyperthyroidism in adults. In: Basow DS, ed. UpToDate. Waltham, MA: UpToDate; 2013.

Category: Endocrine system



51. Which of the following statements about altitude sickness is true?


A) Most people are affected at altitudes between 5,000 and 7,500 ft.


B) Dehydration is rarely an associated condition.


C) The most common symptom is headache.


D) Hydrochlorothiazide is used for prophylaxis.


E) A high carbohydrate diet can help prevent symptoms.

View Answer

Answer and Discussion

The answer is C. As altitude increases, the partial pressure of oxygen decreases. Approximately 20% of people experience symptoms ascending to more than 8,000 ft in less than 1 day, and 80% show some symptoms at altitudes higher than 12,700 ft. Symptoms include headache (most common), impaired concentration, nausea, vomiting, fatigue, dyspnea and hyperventilation, palpitations, and insomnia. Any type of physical exertion usually aggravates the symptoms, and excessive hyperventilation leads to dehydration. In severe cases, pulmonary and cerebral edema can occur. The most common manifestation is acute mountain sickness, heralded by malaise and headache.

Risk factors include young age, residence at a low altitude, rapid ascent, strenuous physical exertion, and a previous history of altitude illness. However, activity restriction is not necessary for those with coronary artery disease (CAD) who are traveling to high altitudes. Acetazolamide (a carbonic anhydrase inhibitor) helps prevent respiratory alkalosis, which contributes to the symptoms. It is an effective prophylactic treatment started one day before ascent and continued until the patient acclimatizes to their final altitude. Dexamethasone is an effective alternative treatment for those with contraindictions to acetazolamide (which includes a sulfa allergy).

Treatment involves hydration and usually only symptomatic measures are necessary.

Additional Reading: Altitude illness: risk factors, prevention, presentation, and treatment. Am Fam Physician. 2010;82(9): 1103-1110.

Category: Respiratory system



52. Anemia that is seen in patients with chronic renal disease is usually caused by insufficient


A) Iron stores


B) Vitamin B12


C) Renin levels


D) Erythropoietin levels


E) Folate stores

Jun 18, 2016 | Posted by in GENERAL & FAMILY MEDICINE | Comments Off on Care of the Adult Patient

Full access? Get Clinical Tree

Get Clinical Tree app for offline access